Find the cardinality of set ${(x,y,z):x,y,z in mathbb{Z}^+ wedge xyz=108}$












0












$begingroup$


Find the cardinality of set ${(x,y,z): x,y,ztext{ are positive integers and }xyz=108}$



my attempt:



I write all factors of $108$.



$$1,2,3,4,6,9,12,18,27,36,54,108.$$
First observe that $xyz $ has only two degree of freedom. So its enough to choose $x,y$ . If I choose $x=1$ there are $12$ ways of choosing $y$. If choose $x$ as $2$ there are $11$ ways ...and so on.



So total number of ways $=1+2+...12=78$.
But its give in my book that $60$ is answer. Where am I wrong?










share|cite|improve this question











$endgroup$








  • 1




    $begingroup$
    "If I choose $2$ there are $11$ ways of choosing $y$" Really? I only found $8$ ways: $1,2,3,6,9,18,27,54$. What are the other $3$ ways???
    $endgroup$
    – bof
    Jan 15 at 7:36










  • $begingroup$
    @bof oh...thanks
    $endgroup$
    – Cloud JR
    Jan 15 at 7:38
















0












$begingroup$


Find the cardinality of set ${(x,y,z): x,y,ztext{ are positive integers and }xyz=108}$



my attempt:



I write all factors of $108$.



$$1,2,3,4,6,9,12,18,27,36,54,108.$$
First observe that $xyz $ has only two degree of freedom. So its enough to choose $x,y$ . If I choose $x=1$ there are $12$ ways of choosing $y$. If choose $x$ as $2$ there are $11$ ways ...and so on.



So total number of ways $=1+2+...12=78$.
But its give in my book that $60$ is answer. Where am I wrong?










share|cite|improve this question











$endgroup$








  • 1




    $begingroup$
    "If I choose $2$ there are $11$ ways of choosing $y$" Really? I only found $8$ ways: $1,2,3,6,9,18,27,54$. What are the other $3$ ways???
    $endgroup$
    – bof
    Jan 15 at 7:36










  • $begingroup$
    @bof oh...thanks
    $endgroup$
    – Cloud JR
    Jan 15 at 7:38














0












0








0





$begingroup$


Find the cardinality of set ${(x,y,z): x,y,ztext{ are positive integers and }xyz=108}$



my attempt:



I write all factors of $108$.



$$1,2,3,4,6,9,12,18,27,36,54,108.$$
First observe that $xyz $ has only two degree of freedom. So its enough to choose $x,y$ . If I choose $x=1$ there are $12$ ways of choosing $y$. If choose $x$ as $2$ there are $11$ ways ...and so on.



So total number of ways $=1+2+...12=78$.
But its give in my book that $60$ is answer. Where am I wrong?










share|cite|improve this question











$endgroup$




Find the cardinality of set ${(x,y,z): x,y,ztext{ are positive integers and }xyz=108}$



my attempt:



I write all factors of $108$.



$$1,2,3,4,6,9,12,18,27,36,54,108.$$
First observe that $xyz $ has only two degree of freedom. So its enough to choose $x,y$ . If I choose $x=1$ there are $12$ ways of choosing $y$. If choose $x$ as $2$ there are $11$ ways ...and so on.



So total number of ways $=1+2+...12=78$.
But its give in my book that $60$ is answer. Where am I wrong?







combinatorics permutations combinations integers






share|cite|improve this question















share|cite|improve this question













share|cite|improve this question




share|cite|improve this question








edited Jan 15 at 7:52









David G. Stork

11k41432




11k41432










asked Jan 15 at 6:59









Cloud JRCloud JR

909518




909518








  • 1




    $begingroup$
    "If I choose $2$ there are $11$ ways of choosing $y$" Really? I only found $8$ ways: $1,2,3,6,9,18,27,54$. What are the other $3$ ways???
    $endgroup$
    – bof
    Jan 15 at 7:36










  • $begingroup$
    @bof oh...thanks
    $endgroup$
    – Cloud JR
    Jan 15 at 7:38














  • 1




    $begingroup$
    "If I choose $2$ there are $11$ ways of choosing $y$" Really? I only found $8$ ways: $1,2,3,6,9,18,27,54$. What are the other $3$ ways???
    $endgroup$
    – bof
    Jan 15 at 7:36










  • $begingroup$
    @bof oh...thanks
    $endgroup$
    – Cloud JR
    Jan 15 at 7:38








1




1




$begingroup$
"If I choose $2$ there are $11$ ways of choosing $y$" Really? I only found $8$ ways: $1,2,3,6,9,18,27,54$. What are the other $3$ ways???
$endgroup$
– bof
Jan 15 at 7:36




$begingroup$
"If I choose $2$ there are $11$ ways of choosing $y$" Really? I only found $8$ ways: $1,2,3,6,9,18,27,54$. What are the other $3$ ways???
$endgroup$
– bof
Jan 15 at 7:36












$begingroup$
@bof oh...thanks
$endgroup$
– Cloud JR
Jan 15 at 7:38




$begingroup$
@bof oh...thanks
$endgroup$
– Cloud JR
Jan 15 at 7:38










2 Answers
2






active

oldest

votes


















3












$begingroup$

$$(2^{j_1} 3^{k_1})(2^{j_2}3^{k_2})(2^{j_3}3^{k_3}) = 108 = 2^2 cdot 3^3$$



where ${ j_i, k_i} ge 0$ and by the Prime Factorization Theorem we have $j_1 + j_2 + j_3 = 2$ and $k_1 + k_2 + k_3 = 3$.



How many ways can you solve those equations, up to equivalent permutations of the factors? $12$



Start with the first (smallest) factor $(2^0 3^0)$ and increment others for the first "row"; then $(2^1 3^0)$ and increment others for the second "row"; then $(2^0 3^1)$ and so on... for the third "row." There is no fourth "row"...



Here are the sets, each in ascending order, themselves ordered:




  • {1, 1, 108}, {1, 2, 54}, {1, 3, 36}, {1, 4, 27}, {1, 6, 18}, {1, 9, 12}

  • {2, 2, 27}, {2, 3, 18}, {2, 6, 9}

  • {3, 3, 12}, {3, 4, 9}, {3, 6, 6}






share|cite|improve this answer











$endgroup$





















    3












    $begingroup$

    $108=2^2 cdot 3^3$, so $x,y,z$ must be of the form $2^i3^j$ such that the sum of the $i$'s (where $0le ile 2$) is $2$ and the sum of the $j$'s (also integers starting from $0$) is $3$. How many solutions do $i_x+i_y+i_z = 2$ and $j_x+j_y+j_z = 3$ have in integers $ge 0$? All combinations of these solutions are the solutions you want.



    There are $binom{n+k-1}{k-1}$ non-negative integer solutions to $x_1+ ldots +x_k =n$ (see wikipedia, e.g.) and so for $3$ variables and $n=2$ we get $binom{2+3-1}{2} = binom{4}{2}=6$ and for $n=3$ we get $binom{5}{2} = 10$. So combined we get $6 times 10=60$ solutions for these independent equations.






    share|cite|improve this answer











    $endgroup$













      Your Answer





      StackExchange.ifUsing("editor", function () {
      return StackExchange.using("mathjaxEditing", function () {
      StackExchange.MarkdownEditor.creationCallbacks.add(function (editor, postfix) {
      StackExchange.mathjaxEditing.prepareWmdForMathJax(editor, postfix, [["$", "$"], ["\\(","\\)"]]);
      });
      });
      }, "mathjax-editing");

      StackExchange.ready(function() {
      var channelOptions = {
      tags: "".split(" "),
      id: "69"
      };
      initTagRenderer("".split(" "), "".split(" "), channelOptions);

      StackExchange.using("externalEditor", function() {
      // Have to fire editor after snippets, if snippets enabled
      if (StackExchange.settings.snippets.snippetsEnabled) {
      StackExchange.using("snippets", function() {
      createEditor();
      });
      }
      else {
      createEditor();
      }
      });

      function createEditor() {
      StackExchange.prepareEditor({
      heartbeatType: 'answer',
      autoActivateHeartbeat: false,
      convertImagesToLinks: true,
      noModals: true,
      showLowRepImageUploadWarning: true,
      reputationToPostImages: 10,
      bindNavPrevention: true,
      postfix: "",
      imageUploader: {
      brandingHtml: "Powered by u003ca class="icon-imgur-white" href="https://imgur.com/"u003eu003c/au003e",
      contentPolicyHtml: "User contributions licensed under u003ca href="https://creativecommons.org/licenses/by-sa/3.0/"u003ecc by-sa 3.0 with attribution requiredu003c/au003e u003ca href="https://stackoverflow.com/legal/content-policy"u003e(content policy)u003c/au003e",
      allowUrls: true
      },
      noCode: true, onDemand: true,
      discardSelector: ".discard-answer"
      ,immediatelyShowMarkdownHelp:true
      });


      }
      });














      draft saved

      draft discarded


















      StackExchange.ready(
      function () {
      StackExchange.openid.initPostLogin('.new-post-login', 'https%3a%2f%2fmath.stackexchange.com%2fquestions%2f3074149%2ffind-the-cardinality-of-set-x-y-zx-y-z-in-mathbbz-wedge-xyz-108%23new-answer', 'question_page');
      }
      );

      Post as a guest















      Required, but never shown

























      2 Answers
      2






      active

      oldest

      votes








      2 Answers
      2






      active

      oldest

      votes









      active

      oldest

      votes






      active

      oldest

      votes









      3












      $begingroup$

      $$(2^{j_1} 3^{k_1})(2^{j_2}3^{k_2})(2^{j_3}3^{k_3}) = 108 = 2^2 cdot 3^3$$



      where ${ j_i, k_i} ge 0$ and by the Prime Factorization Theorem we have $j_1 + j_2 + j_3 = 2$ and $k_1 + k_2 + k_3 = 3$.



      How many ways can you solve those equations, up to equivalent permutations of the factors? $12$



      Start with the first (smallest) factor $(2^0 3^0)$ and increment others for the first "row"; then $(2^1 3^0)$ and increment others for the second "row"; then $(2^0 3^1)$ and so on... for the third "row." There is no fourth "row"...



      Here are the sets, each in ascending order, themselves ordered:




      • {1, 1, 108}, {1, 2, 54}, {1, 3, 36}, {1, 4, 27}, {1, 6, 18}, {1, 9, 12}

      • {2, 2, 27}, {2, 3, 18}, {2, 6, 9}

      • {3, 3, 12}, {3, 4, 9}, {3, 6, 6}






      share|cite|improve this answer











      $endgroup$


















        3












        $begingroup$

        $$(2^{j_1} 3^{k_1})(2^{j_2}3^{k_2})(2^{j_3}3^{k_3}) = 108 = 2^2 cdot 3^3$$



        where ${ j_i, k_i} ge 0$ and by the Prime Factorization Theorem we have $j_1 + j_2 + j_3 = 2$ and $k_1 + k_2 + k_3 = 3$.



        How many ways can you solve those equations, up to equivalent permutations of the factors? $12$



        Start with the first (smallest) factor $(2^0 3^0)$ and increment others for the first "row"; then $(2^1 3^0)$ and increment others for the second "row"; then $(2^0 3^1)$ and so on... for the third "row." There is no fourth "row"...



        Here are the sets, each in ascending order, themselves ordered:




        • {1, 1, 108}, {1, 2, 54}, {1, 3, 36}, {1, 4, 27}, {1, 6, 18}, {1, 9, 12}

        • {2, 2, 27}, {2, 3, 18}, {2, 6, 9}

        • {3, 3, 12}, {3, 4, 9}, {3, 6, 6}






        share|cite|improve this answer











        $endgroup$
















          3












          3








          3





          $begingroup$

          $$(2^{j_1} 3^{k_1})(2^{j_2}3^{k_2})(2^{j_3}3^{k_3}) = 108 = 2^2 cdot 3^3$$



          where ${ j_i, k_i} ge 0$ and by the Prime Factorization Theorem we have $j_1 + j_2 + j_3 = 2$ and $k_1 + k_2 + k_3 = 3$.



          How many ways can you solve those equations, up to equivalent permutations of the factors? $12$



          Start with the first (smallest) factor $(2^0 3^0)$ and increment others for the first "row"; then $(2^1 3^0)$ and increment others for the second "row"; then $(2^0 3^1)$ and so on... for the third "row." There is no fourth "row"...



          Here are the sets, each in ascending order, themselves ordered:




          • {1, 1, 108}, {1, 2, 54}, {1, 3, 36}, {1, 4, 27}, {1, 6, 18}, {1, 9, 12}

          • {2, 2, 27}, {2, 3, 18}, {2, 6, 9}

          • {3, 3, 12}, {3, 4, 9}, {3, 6, 6}






          share|cite|improve this answer











          $endgroup$



          $$(2^{j_1} 3^{k_1})(2^{j_2}3^{k_2})(2^{j_3}3^{k_3}) = 108 = 2^2 cdot 3^3$$



          where ${ j_i, k_i} ge 0$ and by the Prime Factorization Theorem we have $j_1 + j_2 + j_3 = 2$ and $k_1 + k_2 + k_3 = 3$.



          How many ways can you solve those equations, up to equivalent permutations of the factors? $12$



          Start with the first (smallest) factor $(2^0 3^0)$ and increment others for the first "row"; then $(2^1 3^0)$ and increment others for the second "row"; then $(2^0 3^1)$ and so on... for the third "row." There is no fourth "row"...



          Here are the sets, each in ascending order, themselves ordered:




          • {1, 1, 108}, {1, 2, 54}, {1, 3, 36}, {1, 4, 27}, {1, 6, 18}, {1, 9, 12}

          • {2, 2, 27}, {2, 3, 18}, {2, 6, 9}

          • {3, 3, 12}, {3, 4, 9}, {3, 6, 6}







          share|cite|improve this answer














          share|cite|improve this answer



          share|cite|improve this answer








          edited Jan 15 at 7:44

























          answered Jan 15 at 7:19









          David G. StorkDavid G. Stork

          11k41432




          11k41432























              3












              $begingroup$

              $108=2^2 cdot 3^3$, so $x,y,z$ must be of the form $2^i3^j$ such that the sum of the $i$'s (where $0le ile 2$) is $2$ and the sum of the $j$'s (also integers starting from $0$) is $3$. How many solutions do $i_x+i_y+i_z = 2$ and $j_x+j_y+j_z = 3$ have in integers $ge 0$? All combinations of these solutions are the solutions you want.



              There are $binom{n+k-1}{k-1}$ non-negative integer solutions to $x_1+ ldots +x_k =n$ (see wikipedia, e.g.) and so for $3$ variables and $n=2$ we get $binom{2+3-1}{2} = binom{4}{2}=6$ and for $n=3$ we get $binom{5}{2} = 10$. So combined we get $6 times 10=60$ solutions for these independent equations.






              share|cite|improve this answer











              $endgroup$


















                3












                $begingroup$

                $108=2^2 cdot 3^3$, so $x,y,z$ must be of the form $2^i3^j$ such that the sum of the $i$'s (where $0le ile 2$) is $2$ and the sum of the $j$'s (also integers starting from $0$) is $3$. How many solutions do $i_x+i_y+i_z = 2$ and $j_x+j_y+j_z = 3$ have in integers $ge 0$? All combinations of these solutions are the solutions you want.



                There are $binom{n+k-1}{k-1}$ non-negative integer solutions to $x_1+ ldots +x_k =n$ (see wikipedia, e.g.) and so for $3$ variables and $n=2$ we get $binom{2+3-1}{2} = binom{4}{2}=6$ and for $n=3$ we get $binom{5}{2} = 10$. So combined we get $6 times 10=60$ solutions for these independent equations.






                share|cite|improve this answer











                $endgroup$
















                  3












                  3








                  3





                  $begingroup$

                  $108=2^2 cdot 3^3$, so $x,y,z$ must be of the form $2^i3^j$ such that the sum of the $i$'s (where $0le ile 2$) is $2$ and the sum of the $j$'s (also integers starting from $0$) is $3$. How many solutions do $i_x+i_y+i_z = 2$ and $j_x+j_y+j_z = 3$ have in integers $ge 0$? All combinations of these solutions are the solutions you want.



                  There are $binom{n+k-1}{k-1}$ non-negative integer solutions to $x_1+ ldots +x_k =n$ (see wikipedia, e.g.) and so for $3$ variables and $n=2$ we get $binom{2+3-1}{2} = binom{4}{2}=6$ and for $n=3$ we get $binom{5}{2} = 10$. So combined we get $6 times 10=60$ solutions for these independent equations.






                  share|cite|improve this answer











                  $endgroup$



                  $108=2^2 cdot 3^3$, so $x,y,z$ must be of the form $2^i3^j$ such that the sum of the $i$'s (where $0le ile 2$) is $2$ and the sum of the $j$'s (also integers starting from $0$) is $3$. How many solutions do $i_x+i_y+i_z = 2$ and $j_x+j_y+j_z = 3$ have in integers $ge 0$? All combinations of these solutions are the solutions you want.



                  There are $binom{n+k-1}{k-1}$ non-negative integer solutions to $x_1+ ldots +x_k =n$ (see wikipedia, e.g.) and so for $3$ variables and $n=2$ we get $binom{2+3-1}{2} = binom{4}{2}=6$ and for $n=3$ we get $binom{5}{2} = 10$. So combined we get $6 times 10=60$ solutions for these independent equations.







                  share|cite|improve this answer














                  share|cite|improve this answer



                  share|cite|improve this answer








                  edited Jan 15 at 18:17

























                  answered Jan 15 at 7:12









                  Henno BrandsmaHenno Brandsma

                  110k347116




                  110k347116






























                      draft saved

                      draft discarded




















































                      Thanks for contributing an answer to Mathematics Stack Exchange!


                      • Please be sure to answer the question. Provide details and share your research!

                      But avoid



                      • Asking for help, clarification, or responding to other answers.

                      • Making statements based on opinion; back them up with references or personal experience.


                      Use MathJax to format equations. MathJax reference.


                      To learn more, see our tips on writing great answers.




                      draft saved


                      draft discarded














                      StackExchange.ready(
                      function () {
                      StackExchange.openid.initPostLogin('.new-post-login', 'https%3a%2f%2fmath.stackexchange.com%2fquestions%2f3074149%2ffind-the-cardinality-of-set-x-y-zx-y-z-in-mathbbz-wedge-xyz-108%23new-answer', 'question_page');
                      }
                      );

                      Post as a guest















                      Required, but never shown





















































                      Required, but never shown














                      Required, but never shown












                      Required, but never shown







                      Required, but never shown

































                      Required, but never shown














                      Required, but never shown












                      Required, but never shown







                      Required, but never shown







                      Popular posts from this blog

                      Can a sorcerer learn a 5th-level spell early by creating spell slots using the Font of Magic feature?

                      Does disintegrating a polymorphed enemy still kill it after the 2018 errata?

                      A Topological Invariant for $pi_3(U(n))$